RegistrierenRegistrieren   LoginLogin   FAQFAQ    SuchenSuchen   
Welche Konstruktion trägt am meisten Gewicht?
 
Neue Frage »
Antworten »
    Foren-Übersicht -> Mechanik
Autor Nachricht
BrunoBruno
Gast





Beitrag BrunoBruno Verfasst am: 28. Sep 2021 15:25    Titel: Welche Konstruktion trägt am meisten Gewicht? Antworten mit Zitat

Meine Frage:
Liebe Physikergemeinschaft,

anbei der Link zur Aufgabe: https://gyazo.com/16b2eb37381cc60db047bd690ed03e70


Welche Konstruktion trägt am meisten Gewicht, wenn sechs schwere Gegenstände auf dem gesamten Brett verteilt werden?

Vielen Dank für Eure Hilfe.






Meine Ideen:
Ich vermute, dass B die korrekte Antwort ist. Das Brett sollte auf beiden Seiten wahrscheinlich einen leichten Überhang haben. Doch warum ist die Variante mit dem leichten Überhang (sofern dies korrekt ist) die stabilste Variante?
BrunoBruno
Gast





Beitrag BrunoBruno Verfasst am: 28. Sep 2021 17:56    Titel: Kann mir jemand helfen? Antworten mit Zitat

Kann mir hier jemand helfen?
JungeLenz



Anmeldungsdatum: 05.07.2020
Beiträge: 31

Beitrag JungeLenz Verfasst am: 28. Sep 2021 19:30    Titel: Drehmoment Antworten mit Zitat

Ich würde sagen A ist am stabilsten.

Wenn du 6 schwere Gegenstände gleichmäßig verteilst dann würde ja bei B und C das meiste Gewicht in der Mitte liegen, also da ist es wahrscheinlicher dass das Regal in der Mitte durchbricht.

Bei A wird aber das Regal in gleichem Maße in der Mitte und an den Seiten belastet.
Bzw. die Drehmomente sind ausgeglichen.

Du kannst dir auch das hier dazu durchlesen:
https://physics.stackexchange.com/questions/636568/which-two-points-should-a-bookshelf-be-supported-at-to-minimize-maximum-strain

https://www.quora.com/How-do-I-place-the-supports-such-that-the-maximum-bending-moment-is-the-least-possible-in-a-simply-supported-beam
Füsik-Gast
Gast





Beitrag Füsik-Gast Verfasst am: 28. Sep 2021 19:31    Titel: Brückenkräfte Antworten mit Zitat

Die Hebelgesetze kann man auch für den Vergleich der Brückenkräfte verwenden.
Bei A sind die Hebel am kürzesten,daher dort geringste Belastung.
Du kannst zuer Veranschaulichung hier auch einfach nur ein Gewicht in der Balkenmitte positionieren.
Wo würde der Balken am ehesten durchbrechen?

Füsik-Gast.
roycy



Anmeldungsdatum: 05.05.2021
Beiträge: 961

Beitrag roycy Verfasst am: 28. Sep 2021 21:10    Titel: Re: Kann mir jemand helfen? Antworten mit Zitat

BrunoBruno hat Folgendes geschrieben:
Kann mir hier jemand helfen?


Ich finde die Aufgabe etwas unklar. Sie müßte m. E. lauten, welche Konstruktion sich bei "Streckenlast" insgesamt am geringsten durchbiegt.
Für mich ist das A, weil der seitliche jeweilige Überhang ca 25% beträgt.
(Bessel).
Neue Frage »
Antworten »
    Foren-Übersicht -> Mechanik